Difference between revisions of "2006 AMC 12A Problems/Problem 4"

Line 1: Line 1:
 
== Problem ==
 
== Problem ==
 +
 +
A digital watch displays hours and minutes with AM and PM. What is the largest possible sum of the digits in the display?
 +
 +
<math> \mathrm{(A) \ } 17\qquad \mathrm{(B) \ } 19\qquad \mathrm{(C) \ } 21\qquad \mathrm{(D) \ } 22</math>
 +
 +
<math>\mathrm{(E) \ }  23</math>
  
 
== Solution ==
 
== Solution ==

Revision as of 23:41, 10 July 2006

Problem

A digital watch displays hours and minutes with AM and PM. What is the largest possible sum of the digits in the display?

$\mathrm{(A) \ } 17\qquad \mathrm{(B) \ } 19\qquad \mathrm{(C) \ } 21\qquad \mathrm{(D) \ } 22$

$\mathrm{(E) \ }  23$

Solution

See also